Family Medicine I - All Questions For Final and Midterm
Family Medicine I - All Questions For Final and Midterm
PREVENTION: __________________________________________________________________________ 1
PREVENTIVE ACTIONS ____________________________________________________________________ 5
PRENATAL CARE ________________________________________________________________________ 9
CHEST PAIN: _________________________________________________________________________13
CARDIOVASCULAR: _____________________________________________________________________13
ATRIAL FIBRILLATION ____________________________________________________________________16
MYOCARDIAL INFARCTION : _______________________________________________________________ 17
CARDIOVASCULAR DRUGS: ________________________________________________________________ 21
HYPERTENSION ________________________________________________________________________22
DIABETES ____________________________________________________________________________30
DIABETES DRUGS ______________________________________________________________________32
THYROID ____________________________________________________________________________33
ABDOMINAL PAIN ______________________________________________________________________39
DYSPEPSIA ___________________________________________________________________________44
LIVER ______________________________________________________________________________46
LOWER INTESTINAL _____________________________________________________________________49
ACUTE RESPIRATORY ____________________________________________________________________52
ADDITIONAL QUESTIONS: ________________________________________________________________ 61
Prevention:
1. Which of the following statements is true according to prevention:
A. Primary Prevention: early diagnostics of pre-clinical stage of a disease
B. Secondary Prevention: disease prevention- avoiding development of a disease;
C. Tertiary Prevention: prevention of complications, disability
D. All of above
6. Which of the following statements are true according to primary health care?
A. Primary health care is the urgent care only
B. Primary health care is the management of acute problems only
C. Primary health care is the key to attaining health for all
D. Primary health care is the prevention, not the management of the diseases
Ahmad Soboh 2
7. All of the following statements are true according to primary health care, except:
A. Primary care is performed and managed by a personal physician who is
specialized in family medicine
B. Primary care promotes effective communication with patients and
encourages the role of the patient as a partner in health care.
C. Primary care is that care provided by physicians specifically trained for and
skilled in comprehensive first contact and continuing care for persons with
any undiagnosed sign, symptom, or health concern.
D. Primary care includes diagnosis and treatment of acute and chronic illnesses
in a variety of health care settings and does not cover the prevention
8. All of the following statements are true according to levels of health care, except
A. Primary care is urgent care only
B. Secondary Care is offered by various specialists (other than GP / FD) at the
hospitals (In-patient care)
C. Tertiary Care is High technology, highly specialized care offered at
specialized centers and university clinics
D. Extended Care ensures rehabilitation and long term care to elderly people, to
people with chronic illness and disability within special network of institutions
9. Which of the following is true about the characteristic of the discipline of FM/GP:
A. Normally the point of first medical contact within the health care system,
providing open and unlimited access to its users, dealing with all health problems
regardless of the age, sex, or any other characteristic of the person concerned.
B. Deals with health problems in their only medical dirnensions:
C. Manages only chronic health problems of individual;
D. Manages only acute health problems of individual:
10. All of the following is true about the competencies of the discipline of FM/GP.
A. Person Centred Care
B. Holistic approach
C. Primary Care Management
D. Comprehensive approach
E. All of above
16. Which of the following screening test's characteristics is necessary for effective screening?
A. Validity and reliability (sensitivity and specificity)
B. Simple, easily acceptable
C. Acceptable for patients
D. Inexpensive
E. All of the above:
17. Which of the following characteristics of disease is nessesary for effective screening:
A. The disease must have long lasting pre-clinical phase;
B. The disease must be common (prevalence of the disease);
C. Availability of effective treatment on pre clinical phase;
D. Morbidity and mortality, frequency of fatal outcome;
E. All of the above;
19. All of the statements about open questions are correct, except:
A. Open questions encourage a patient to formulate his own thoughts,
feelings, desires, requests
B. Open questions are questions with the answer “yes” or “no”
C. Open questions are very important during initial stage
D. Open questions help in determining the main reason of the visit
22. You are working with a patient who recently retired from his job after 41 years. He is
currently 67 years old and has hypertension and high cholesterol. His wife thinks he
drinks too much, and during your visit, he admits to 3 alcoholic beverages per day. You
have screened him for alcoholism, and he does not meet the criteria. You would like to
negotiate a safe drinking amount for this patient. Which of the following best
represents a safe level of alcohol intake for this patient?
A. Seven drinks per week, no more than 3 per occasion.
B. Fourteen drinks per week, no more than 4 per occasion.
C. No more than 1 drink per day.
D. No more than 2 beers or glasses of wine per day or no more than 1
alcoholic beverage per day.
E. There is no safe drinking amount for this patient
→ Primary prevention for healthy lifestyle – For patients older than age 65, it is recommended that they ingest no
more than 1 drink per day.
Preventive Actions
24. An 18 year old women comes for health evaluation. Which preventive actions
are taken in this case? P.62
A. Colorectal cancer screening
B. Physical examination
C. Mammography
D. Lipid profile
25. 27 years old worn admits for health evaluation. Which preventive actions are
shown in this case?
A. Colonoscopy
B. Mammography
C. PAP test
D. Echocardiography
26. A 30-year-old man who weighs 170 pounds at a height of 5 ft 10.5 in. has a
repeatable blood pressure in the 110–130/70–75 range. His family history reveals
that his parents are living and well at 50 and 55 years of age. He wishes to know
when he should be screened for “cholesterol.” You reply that according to the U.S.
Preventive Services Task Force that should begin at what age? NMS
A. 25 years
B. 30 years
C. 35 years
D. 45 years
E. 50 years
→ Begin at 35 years of age screening men for cholesterol and other lipids, assuming no increase
in risk for atherosclerotic disease (previous heart disease, hypertension, diabetes);
27. 49 years old woman admits for health evaluation. Which preventive actions
are shown in this case except:
A. Mammography (50<)
B. Calculate BMI (at first or if she obesity)
C. Colonoscopy (45-75 years)
D. Lipid profile (45<women)
E. Pap test (21<)
28. You are discussing cancer screening with a patient. Her father was diagnosed
with colorectal cancer at age 62. When should you recommend she begins
colorectal cancer screening?
A. 40 years
B. 50 years
C. 52 years
D. 60 years
E. 62 years
Ahmad Soboh 6
29. You are discussing cancer screening with a female patient. She has no family
history of breast cancer, and routine risk analysis indicates that she is not at
increased risk for the disease. According to the US Preventive Services Task
Force (USPSTF), at what age should she start getting routine mammograms
A. 30 years
B. 40 years
C. 45 years
D. 50 years
E. 60 years
→ You are discussing cancer screening with a female patient. She has no family history of breast cancer, and routine
risk analysis indicates that she is not at increased risk for the disease. According to the US Preventive Services Task
Force (USPSTF), at what age should she start getting routine mammograms
30. You are consulting with a 50-year-old man at the time of his first comprehensive
physical examination in 15 years. You have on hand a baseline electrocardiogram
(EKG) taken when he had his last examination 15 years ago, read as normal (another
was unchanged during an emergency department visit for atypical chest pain 1 year
ago). He expresses a desire to begin preventive health care under your guidance.
Along the way, you elicit information that he smokes one pack of cigarettes per day,
weighs 208 lb (94.2 kg) at a height of 5 ft 10 in. (1.78 m), and has a history of “high
cholesterol.” Which of the following is the most appropriate combination of
screening tests for this person, in addition to routine physical examination? NMS
A. EKG, complete blood cell (CBC) count, and lipid screen
B. EKG stress test, CBC count, and liver and kidney function studies
C. Chest x-ray, EKG, and comprehensive chemical profile
D. Prostate-specific antigen (PSA) test, lipid screen, fecal occult blood test
(FOBT), and blood sugar test
E. Chest x-ray, pulmonary function studies, lipid screen and routine
chemistries, and CBC count
31. The U.S. Preventive Services Task Force (USPSTF) recommends which of the
following screening tests for the listed patients?Harrison’s
A. 38-year-old prior smoker: ultrasound for abdominal aortic aneurysm
B. 45-year-old sexually active woman: nucleic acid amplifcation on a cervical
swab for chlamydia
C. 50-year-old woman with a smoking history: dual-energy x-ray
absorptiometry (DEXA) scan for osteoporosis
D. 80-year-old man: anti-hepatitis C virus (HV) antibody for hepatitis C
E. Non of the above
32. You are consulting a 52-year-old obese man. He is a smoker. Which of the
following is the most appropriate screening test for this person? NMS
A. ECG, complete blood cell (CBC) count, and lipid screen, Creatinine
B. Chest x-ray, ECG, and comprehensive chemical profile
C. Lipid screen, fecal occult blood test (FOBT), and blood sugar test
D. Chest x-ray, pulmonary function studies, lipid screen and routine
chemistries, and CBC count
Ahmad Soboh 7
33. 53 years old woman admits for health evaluation. Which preventive actions
are shown in this case?
A. PAP test
B. Mammography
C. Colonoscopy
D. All of the above
34. 55 years old man admits for health evaluation. Which preventive actions are
shown in this case except
A. Abdominal US for Abdominal aortic aneurysm
B. Ask about physical activities
C. Ask about alcohol use
D. Calculate BMI
E. Lipid profile
→ Abdominal US (ultrasound) for Men aged 65 to 75 years who have ever smoked.
35. You are seeing a 58-year-old smoker for a routine health examination. You
have counseled him on discontinuing tobacco use, and he is considering that
alternative. He denies coughing, shortness of breath, or hemoptysis. Which of
the following is a recommended screen for lung cancer in this patient? Pre-Test
A. He should not be screened for lung cancer.
B. Chest x-ray.
C. Chest computed tomography (CT).
D. Sputum cytology.
E. Bronchoscopy.
36. 61 years old woman admits for health evaluation. Which preventive actions
are shown in this case?
A. Mammography
B. DEXA scan
C. Colonoscopy
D. All of the above
37. A 52-year-old woman with a history of hypertension and diabetes comes to the
office for review of preventive medicine recommendations. She had a mammogram
at age 50 but has had no other preventive medicine tests or advice that she can
recall. She does not smoke cigarettes, does not drink alcohol, and is overweight
(BMI = 29). Her mother developed breast cancer at age 75 and her father had colon
cancer detected at age 65. According to the U.S. Preventive Services Task Force,
which combination of screening tests should you recommend
A. Screening mammography, a chest x-ray, colonoscopy, a screening aortic sono-gram,
and a bone density scan
B. Screening mammography, Pap smear, and colonoscopy
C. Screening mammography, a chest CT scan, colonoscopy, and a bone density scan
D. Screening mammography, a CA-125 blood test, Pap smear, and a bone density scan
E. Screening mammography, colonoscopy, and a bone density scan
Ahmad Soboh 8
38. A 75-year-old G2P2 presents to your gynecology office for a routine examination. She
tells you that she does not have an internist and does not remember the last time she
had a physical examination. She says she is very healthy and denies taking any
medication, including hormone replacement therapy. She has no history of abnormal
Pap smears. She is a nonsmoker and has an occasional cocktail with her dinner. She
does not have any complaints. In addition, she denies any family history of cancer. The
patient tells you that she is a widow and lives alone in an apartment in town. Her
grown children have families of their own and live far away. She states that she is self-
sufficient and spends her time visiting friends and volunteering at a local museum. Her
blood pressure is 140/70 mm Hg. Her height is 5 ft 4 in and she weighs 130 lb. Her
physical examination is completely normal. Which of the following are the most
appropriate screening tests to order for this patient? PreTest- Gyno
A. Pap smear and mammogram
B. Pap smear, mammogram, and colonoscopy
C. Mammogram, colonoscopy, and bone densitometry
D. Mammogram, colonoscopy, bone densitometry, and TB skin test
E. Mammogram, colonoscopy, bone densitometry, TB skin test, and auditory testing
39. A 21-year-old woman comes to the clinic for a health check-up. She has been
sexually active since she was 17 years old and has never seen a gynecologist before.
The physician discusses the importance of health screening with the patient and
performs a Papanicolaou (or Pap) smear during the visit. Which of the following is
the most appropriate screening schedule recommendation? First Aid – CK
A. Pap tests every 3 years beginning approximately 3 years after first
intercourse or at 21 years of age
B. Yearly Pap test and human papillomavirus DNA testing beginning
approximately 3 years after first intercourse or at 21 years of age
C. Yearly Pap tests beginning approximately 3 years after first intercourse
or at 21 years of age
D. Yearly Pap tests beginning at menarche and ending at menopause
E. Yearly Pap tests once a patient is sexually active or at 21 years of age
40. You are seeing a healthy 26-year-old woman for a routine health visit. She
mentions that she and her husband are thinking about starting a family soon.
She has never been pregnant before. Which of the following interventions, if
done prior to pregnancy, has been shown to have a clear beneficial outcome for
this woman and her potential child? P.37 Pre-Test
A. Blood typing and antibody testing
B. Screening for HIV
C. Screening for Chlamydia
D. Prescribing 0.4 to 0.8 mg of folic acid daily
→ Of the interventions listed above, only prescribing folic acid has been shown to be beneficial prior to pregnancy.
It will decrease the chance of neural tube defects in the baby. The other interventions should be done early in the
pregnancy to ensure good pregnancy outcome.
Ahmad Soboh 9
41. 65 years old men admits for health evaluation. She denies any complains. All of
the following preventive actions are shown in this case except?
A. Lipid test;
B. Lungx-ray
C. Colonoscopy
D. Fecal occult blood test;
E. PSA:
Prenatal Care
42. A 28-year-old woman with a history of epilepsy presents for a preconception
consultation visit. Which of the following is the most important advice to give
to this patient? Case-File
A. Diabetes screening prior to pregnancy.
B. EEG reading that is normal prior to conception.
C. Preconception folate supplementation.
D. Stop epilepsy medication prior to pregnancy and through the first trimester.
→Women with a history of epilepsy should receive 1 mg of folic acid supplementation daily to help prevent neural tube
defects. In general epilepsy medications should be continued, although the type of medication may be changed. For
instance, valproic acid has a relatively high rate of neural tube defects associated with its use, and if possible, another
medication should be used.
43. All of the following preventive actions are shown in firs trimester prenatal care
except: P.42
A. Glucose tolerance test (blood glucose measured 1 hour after oral
ingestion of 50 g glucose in 150 mL fluid)*
B. Hepatitis B surface antigen (HbsAg);
C. Hemoglobin/hematocrit
D. Gonorrhea/Chlamydia
E. Rubella titer
45. A 25-year-old primigravida presents to the obstetrician’s offi ce for her fi rst prenatal
visit. Which of the following titers is routinely obtained at the fi rst prenatal visit?
A. Cytomegalovirus and hepatitis B
B. Cytomegalovirus and rubella
C. Rubella and syphilis
D. Rubella, syphilis, HIV, hepatitis B, cytomegalovirus, and toxoplasmosis
E. Toxoplasmosis and HIV
Ahmad Soboh 10
46. A 40-year-old woman pregnant at 6 weeks gestation presents to your office for prenatal
care. She is interested in prenatal testing for genetic abnormalities. She read on the
Internet that an ultrasound measurement of the neck of the fetus can be used in prenatal
diagnosis. Which of the following is correct information to tell your patient regarding
ultrasound measurement of the fetal nuchal translucency for prenatal diagnosis? PreTest- Gyno
A. It is a simple way to screen for Turner syndrome.
B. It can be performed by anyone trained in basic fetal ultrasonography.
C. It should be offered only to pregnant women less than 35 years.
D. It can be performed at any gestational age.
E. It is a screening test for Down syndrome performed between 10 and 13
weeks of pregnancy.
→ Nuchal translucency (NT) of the fetal neck using ultrasound is often combined with β-HCG
and PAPP-A for first trimester Down syndrome screening (called first trimester combined test).
NT is the measurement of the subcutaneous space between the skin and the cervical spine of
the fetus. Fetuses with increased nuchal thickness may be at risk for Down syndrome. P.43
47. A patient who works as a nurse in the surgery intensive care unit at a local
community hospital comes to see you for her annual gynecologic examination.
She tells you that she plans to go off her oral contraceptives because she plans
to attempt pregnancy in the next few months. This patient has many questions
regarding updating her immunizations and whether or not she can do this when
pregnant. Which of the following is the most appropriate recommendation?
A. The patient should be checked for immunity against the rubella (German
measles) virus prior to conception because the rubella vaccine contains a
live virus and should not be given during pregnancy.
B. The patient should be given the tetanus toxoid vaccination prior to becoming
pregnant because it is a live virus vaccine that has been associated with multiple
fetal anomalies when administered during pregnancy.
C. The Centers for Disease Control and Prevention recommends that all pregnant
women should be vaccinated against the influenza virus during the first trimester.
D. If she is exposed to chicken pox while she is pregnant she can be immunized at
that time since the chicken pox vaccine is safe during pregnancy.
E. Because of her occupation, the patient is at high risk for hepatitis B; she should
complete the hepatitis B vaccination series before she conceives, since that
vaccine has been associated with neonatal jaundice.
48. A 69-year-old man approaches a physician to ask about his target cholesterol
level. He has history of coronary arteries disease. He does not smoke, denies
diabetes, and has a body mass index of 26 kg/m2. The man says he has heard
there are multiple types of cholesterol, but wants to know just one marker that
will be the best for him to follow. The physician advises that following the LDL
cholesterol would be a reasonable option. At what LDL cholesterol level should
be considered for this patient?First Aid – CK
A. LDL cholesterol >100 mg/dL
B. LDL cholesterol >160 mg/dL
C. LDL cholesterol >190 mg/dL
D. LDL cholesterol >200 mg/dL
Ahmad Soboh 11
49. A 54-year-old man visits his physician for a regular check-up. The man has a
history of diverticular disease and diabetes and has no current medical
complaints. He denies tobacco use or any family history of cardiac disease. On
examination, the patient is afebrile and normotensive. His lipid profi le shows a
total cholesterol level of 230 mg/dL, LDL cholesterol of 135 mg/dL, and HDL
cholesterol of 47 mg/dL. In terms of managing this man’s hyperlipidemia, what
is the best next step?
A. Lower LDL cholesterol to <100 mg/dL
B. Lower LDL cholesterol to <130 mg/dL
C. Raise HDL cholesterol to >50 mg/dL
D. Recommend a high-fi ber and low-fat diet
E. Recommend regular exercise
50. You send the patient to the laboratory for some screening tests. All of the
following are appropriate tests to order in this patient except
A. Lipid profile
B. CA-125
C. TSH
D. Urinalysis
E. Fasting blood sugar
→ Women over 65 years old should undergo cholesterol testing every 3 to 5 years, fasting glucose testing every 3
years, screening for thyroid disease with a TSH every 3 to 5 years, and periodic urinalysis. CA-125 testing is not
recommended for ovarian cancer screening in women who are at low risk for ovarian cancer.
52. Which of the following is a contraindication to receiving the live rubella vaccine Lange
A. children between 1 year old and puberty
B. infants <1 year old
C. pregnant women
D. all exposed patients
E. all adults
→ Pregnancy or anticipated pregnancy within 4 weeks of vaccination is a contraindication to receiving the rubella vaccine.
Ahmad Soboh 12
53. Which of the following is true about well Adult Care-Immunizations and
chemoprophylaxis except: P.69
A. Pneumococcal polysaccharide vaccine (PPV) is indicate for 65 years old pacients
and older
B. Pneumococcal polysaccharide vaccine (PPV) is indicate for pacient after 45
years of pacient and older
C. Tetanus, diphtheria acellular pertussis (TdaP/Td)- All adults 18 age 65 (TdaP) 65
and older (Td), Booster every 10 years
D. Influenza vaccine (including H1N1)- All patients greater than 6 months of age (CDC
recommendation). In particular, high risk groups include: Nursing home/institutional
residents • Chronic disease • Pregnancy, second and third trimester
E. All of the following
54. A 5-year-old child is brought into the pediatrician's office for immunization and
physical examination. The mother is concerned that her child is a little under the
weather': Which of the following is a contraindication to vaccinating the child? Case-File
A. Acute otitis media with a temperature of 1 00°F requiring antibiotic therapy
B. Previous vaccination reaction that consisted of fever and fussiness that lasted
for 2 days
C. History of an allergic reaction to penicillin
D. Previous vaccination reaction that consisted of wheezing and hypotension
56. A 44-year-old G6P3215 presents for her well-woman examination. She tells you
that all of her deliveries were vaginal and that her largest child weighed 2900 g
at birth. How many full-term pregnancies did this patient have? PreTest- Gyno
A. 1
B. 2
C. 3
D. 5
E. 6
→ GTPALC
• G for Gravity, any pregnancy
• T for term pregnancies, over 37 weeks
• P for preterm pregnancies, 22-37 weeks
• A for abortions, spontaneous or induced <22 weeks
• -If medical reason for induced or if elective
• L for living children
• C for complications mother and child
Ahmad Soboh 13
Common Problems
Chest Pain:
57. Which of the following are Red Flags Suggesting Life-threatening Disease in
Patients with Chest Pain: P.104
A. Electrocardiogram changes, especially ST segment elevation or new left
bundle-branch block;
B. Hypotension, poor tissue perfusion, pulmonary edema, or oliguria
C. Tachycardia, tachypnea, hypoxia
D. All of the above
Cardiovascular:
58. The classical risk factors for CAD according to INC-7 and ESH/ESC
recommendations include: P.116
A. Hypertension;
B. Smoking;
C. Family history of early CAD;
D. Hyperlipidernia;
E. All of the above
59. A 47-year-old woman with a body mass index (BMI) of 37 kg/m2 was recently
diagnosed with type 2 diabetes mellitus. As part of her patient education, you
inform her that which of the following is the most common cause of death in
adults with type 2 diabetes mellitus Harrison’s
A. Coronary artery disease
B. Infection
C. Neuropathy
D. Renal failure
E. Stroke
→ Diabetes mellitus, both insulin- and non–insulin-dependent, is an independent risk factor for CAD and accounts
for 14%–50% of new cases of cardiovascular disease.
60. You are seeing a 51-year-old man with a known history of hypercholesterolemia
and hypertension. He reports having a chest pressure, described as "heaviness” in
the substernal area. It is n intermittently throughout the day. Which of the
following is the best way to describe what the patient is feeling? Pre-Test
A. Atypical nonanginal pain
B. Classic angina
C. Nonanginal pain
D. Atypical angina
→ Atypical angina occurs when the patient experiences pain that has the quality and characteristics of angina, or
occurs with exertion, but not both.
Ahmad Soboh 14
62. You are seeing a 44-year-old woman with a known history of asthma who has
had recent complaints of chest pain. She reports a stabbing pain that seems to
be worse with inspiration. It is not associated with activity, but will occur
intermittently throughout the day. Which of the following is the best way to
describe what the patient is feeling?
A. Classic angina
B. Atypical angina
C. Anginal equivalent
D. Nonanginal pain
E. Atypical nonanginal pain
→ Nonanginal pain has neither the quality nor the precipitating features of angina. Typical descriptive terms of
nonanginal pain include “stabbing,” “shooting,” “knifelike,” “jabbing,” and “tingling.”
63. Which of following doesn't belong to CVD risk factors according to INC-7 and
ESH/ESC recommendations?
A. Smoking;
B. Diabetes mellitus
C. Alcohol consumption
D. Age male > 55 years; female > 65 years;
Which of following doesn't belong to CVD risk factors according to INC-7 and
ESH/ESC recommendations?
A. Smoking;
B. Dyslipidemia
C. Diabetes mellitus
D. BMI > 25
64. A 67-year-old smoking woman with known chronic obstructive lung disease and
right ventricular hypertrophy has been treated with intermittent use of a beta-
adrenergic agonist drug. She now has a bout of increased coughing and
shortness of breath. Which of the following blood tests might be the best
indicator ruling out dyspnea caused by heart failure? NMS
A. Serum aldosterone level
B. Serum cortisol level
C. Serum pro-BNP level
D. Serum digoxin level
E. Serum creatinine level
Ahmad Soboh 15
65. You have diagnosed a 59-year-old woman with heart failure. She has a history of
hypertension, but has never had heart failure before. Which of the following tests is
routinely indicated in the initial evaluation of a person with a diagnosis of heart failure?
A. pharmacology stress test
B. Left heart catheterization
C. Treadmill stress test
D. BNP
→ In patients with pre-existing pulmonary disease, B-type natriuretic peptide (BNP or pro-BNP) testing in the emergency
department is effective at distinguishing an exacerbation due to heart failure from that caused by pulmonary disease. P.611
→ Routine laboratory testing in a person with the new diagnosis of heart failure includes an electrocardiogram, a CBC, a
urinalysis, serum creatinine, potassium and albumin levels, and thyroid function studies. An echocardiogram is imperative to
help identify structural abnormalities of the heart and to measure the ejection fraction. Holter monitoring is not routinely
warranted, as it would not identify a cause for heart failure, but would be used to identify an arrhythmia. Catheterization or
stress testing may be important if ischemia or ischemic cardiomyopathy is identified as a cause, but is not a routine initial test.
66. You are seeing a patient who was discharged from the hospital. She initially
presented to the emergency room with dyspnea and was found to be in CHF.
They admitted her for diuresis and initiation of appropriate first-line therapy.
Since being released, she reports that she is comfortable at rest, but that
ordinary activity results in mild dyspnea. According to the New York Heart
Association (NYHA) functional classification, which class of heart failure best
describes this patient?
A. Class I
B. Class II
C. Class III
D. Class IV
E. Class V
67. A 68-year-old man with a history of myocardial infarction and congestive heart
failure is comfortable at rest. However, when walking to his car, he develops
dyspnea, fatigue, and sometimes palpitations. He must rest for several minutes
before these symptoms resolve. Which of the following is his New York Heart
Association classifcation? Harrison’s
A. Class I
B. Class II
C. Class III
D. Class IV
68. You have diagnosed a 69-year-old woman with heart failure. She has a history
of hypertension, but has never had heart failure before. Which of the following
tests is routinely indicated in the initial evaluation of a person with a new
diagnosis of heart failure?
A. Echocardiogram
B. Holter monitor
C. Left heart catheterization
D. Pharmacologic test
69. An 81-year-old man is hospitalized for acute onset of shortness of breath and
lower extremity edema. Although he lives by himself, it is very diffi cult for him
to move around his apartment without experiencing fatigue. He has not seen
his physician in years but was told in the past that he had high blood pressure.
On physical examination his jugular venous pulse is palpated 9 cm above his
sternal notch, inspiratory crackles are heard at his lung bases, and there is 3+
lower extremity edema.
A. Cardiac angiography
B. Echocardiography
C. Endomyocardial biopsy
D. Pulmonary function tests
E. X-ray of the chest
→ The patient most likely has an acute CHF exacerbation with the underlying etiology being hypertension.
→ Echocardiography is an essential test in all patients with newly diagnosed heart failure and is an excellent,
noninvasive method of assessing chamber size, function, and ejection fraction.
Atrial fibrillation
70. Which of the following ECG abnormalities are shown in patient with Atrial
Fibrillation: P.122
A. Absence of P waves, regular ORS complexes
B. Absence of P waves, irregular chaotic QRS complexes
C. Absence P waves, irregular chaotic T waves
D. Regular P waves, irregular chaotic T waves
E. Regular P waves, irregular chaotic QRS complexes
71. You are evaluating a -year69-old man in the office who is complaining of irregular
heart beat. Given his history and risk factors, you are concerned about Atrial fibrilation.
Which of the following ECG features, if present, would suggest your dignosis?
A. Any ST-segment elevation greater than or equal to 1 mm
B. Absence of the P waves
C. T wave inversion
D. Q waves
E. Regular P waves, irregular chaotic T waves;
Ahmad Soboh 17
73. You are seeing 57-year-old woman who is complaining of a “fluttering in her
chest.” She is otherwise well, and denies shortness of breath or other acute
symptoms. On examination, her pulse rate is rapid and irregular. Which of the
following is her most likely diagnosis?
A. Atrial fibrillation
B. Paroxysmal supraventricular tachycardia (PSVT)
C. Stable ventricular tachycardia
D. Stimulant abuse
E. Hyperthyroidism
74. A 51-year-old man presents to your office and complains of palpitations and
shortness of breath. After further questioning, he admits to heavy alcohol
consumption the previous evening. On examination, he is found to have an
irregular heartbeat of 115 bpm. The most likely diagnosis is Bratton’s
A. Ventricular tachycardia
B. Premature ventricular contractions (PVCs)
C. Atrial fibrillation
D. Wolff-Parkinson-White syndrome
Myocardial Infarction :
75. Which of the following is NOT considered a risk factor for MI? Bratton’s
A. Alcoholism
B. Homocystinemia
C. Type A personality
D. Male sex
E. Obesity
Risk factors for MI include:
• Hypertension • Hyperlipidemia—particularly high total cholesterol, high LDL cholesterol, and low high-density
lipoprotein (HDL) cholesterol • Cigarette smoking • Diabetes mellitus • Obesity (increased weight for height) •
Male gender • Family history of CAD • Sedentary lifestyle • Type A personality • Increased age • Postmenopausal •
Homocystinemia
Ahmad Soboh 18
76. A 49-year-old male smoker comes to his family doctor 7 days after a 4-hour bout of
squeezing anterior chest pain that occurred while he was away with a group on a
hunting trip, on which they traveled by canoe for 5 days away from their
automobiles. He thought that he had a bout of heartburn; he has felt well since the
attack and made the appointment “just in case.” His electrocardiogram (ECG) shows
Q waves in leads II, III, and AVF but no ST deviation nor clearly abnormal T-wave
patterns. There is no prior ECG with which to compare. Which of the following
available laboratory tests would be the most sensitive and specific indicator of a
myocardial infarction (MI) having occurred 7 days before?
A. Aspartate amino transferase (AST, SGOT)
B. Myoglobin
C. Treponin
D. Creatine phosphate (CK-MB fraction)
77. A 49-year-old male smoker comes to his family doctor 7 days after a 4-hour bout of
squeezing anterior chest pain that occurred during exercise. His electrocardiogram
(ECG) shows Q waves in leads II, IIl, and AVF but no ST deviation. Which of the
following available laboratory tests would be the most sensitive and specific
indicator of a myocardial infarction (MI) having occurred 7 days before?
A. Aspartate amino transferase (AST, SGOT)
B. Creatine phosphate (CK-MB fraction)
C. Myoglobin
D. Troponin
E. Non of the above
→ Treponin, for which lab study is most sensitive for picking up a 7-day-old MI. Treponin level peaks in 12 hours and lasts 15 days.
78. You are evaluating a patient with chest pain in the emergency department. The
ECG shows no Q waves or ST-segment changes. You wish to know whether the
patient is suffering a myocardial infarction. Which of the following biomarkers is
the preferred biochemical marker of myocardial infarction
A. Creatine kinase
B. CK-MB band
C. Erythrocyte sedimentation rate
D. Lactate dehydrogenase
E. Troponin I
79. You are evaluating a 61-year-old man in the office who is complaining of chest
pain. Given his history and risk factors, you are concerned about myocardial
ischemia, and order an ECG. Which of the following ECG features, if present, would
most markedly increase the likelihood of an acute myocardial infarction?
A. Any ST-segment elevation greater than or equal to 1 mm
B. Any ST-segment depression
C. Any Q wave
D. Any conduction defect
E. New conduction defect
Ahmad Soboh 19
80. You are evaluating a 69-year-old man in the office who is complaining of chest
pain. Given his history and risk factors, you are concerned about myocardial
ischemia. Which of the following ECG features, if present, would most markedly
increase the likelihood of an acute myocardial infarction?
A. Any ST-segment elevation greater than or equal to 1 mm
B. Absence of the P vawes
C. Any conduction defect
D. long QT
83. A 75-year-old man comes into the emergency department with a 10-minute
history of crushing substernal chest pain radiating to his left arm. This man is
well known to the staff due to his long history of chest pain. His creatine
phosphokinase level is elevated and his troponin T level is 0.4 g/mL. Which of
the following is the most likely diagnosis? First Aid – CK
A. Stable angina
B. Hypochondriasis
C. Unstable angina
D. Prinzmetal's angina
E. Acute myocardial infarction
→ This man has a cardiac enzyme leak of specific myocardial markers that suggests acute MI.
Ahmad Soboh 20
84. You are evaluating a 40-year-old male patient in the office who is complaining
of chest pain. His father had a myocardial infarction at age 42, and the patient
is quite concerned. Which characteristic, if included in the history, decreases
the likelihood that his chest pain is cardiac in origin? Pre-Test
A. The pain is worse with inspiration.
B. The pain radiates to his right arm.
C. The pain radiates to his left arm.
D. The pain is associated with nausea.
E. The pain is associated with sweatiness.
87. Mr. Gilotra is a 57-year-old oil rig worker with a history of hypertension,
tobacco abuse, and diabetes mellitus. He presents to the emergency
department with 30 minutes of crushing substernal chest pain radiating to the
jaw and associated with profuse sweating and shortness of breath. His blood
pressure is 115/90 mmHg, and heart rate is 95 bpm. What diagnostic test will
provide the most rapid method of altering therapeutic management for Mr.
Gilotra?
A. 12-lead ECG
B. Coronary CT angiography
C. Echocardiogram
D. Serum creatine kinase-MB (CK-MB) band level
E. Serum troponin I level
Cardiovascular drugs:
88. A 55 year-old-man has pain in his left leg during walking which began 6 month ago. He
does not have chest pain, shortness of breath, palpitations or syncope. Medical history
is unremarkable. He takes no medications. He is an ex-smoker with a 20 pack-year
history. Patient does not use alcohol or illicit drugs. His father died at age 65 from MI.
BP is 147/95mm, pulse 83 regular. Cardiac, pulmonary and abdominal examination are
within normal limits. Peripheral pulses are decreased in the left leg. Ankle-brachial
index is 0.65 in the left leg and 1,1 in the right. ECG-normal. Fasting glucose-normal,
Cholesterol 220 mg/dl, HDL 50 mg/dI, LDL 135mg/dl, Triglycerides 150mg/dl. Which is
the most appropriate next step in this patient? U-world
A. Surgical revascularization
B. Aspirin alone
C. Aspirin and statin
D. Warfarin
A supervised graded exercise program is the most useful intervention to improve functional capacity and reduce
symptomatic claudication in patients with peripheral arterial disease. Antiplatelet agents (eg, aspirin, clopidogrel) reduce
overall cardiovascular mortality. Lipid-lowering therapy with statins should also be given to all patients with clinically
significant atherosclerotic cardiovascular disease.
91. You have diagnosed a 49-year-old man with CHF because of left ventricular systolic
dysfunction. In addition to acute diuresis, which of the following is the best list-line
agent to use for treatment, in the absence of contraindications? Pre-Test
A. ACE inhibitors
B. β-Blockers
C. Calcium channel blockers
D. Nitrates
E. Hydralazine
→ Many clinical trials have shown that ACE inhibitors decrease symptoms, improve quality of life, decrease
hospitalizations, and reduce mortality in patients with NYHA class II to IV heart failure. In addition, they slow the
progression to heart failure among asymptomatic patients with left ventricular systolic dysfunction.
92. Which of the following combinations would be best utilized in the treatment of
diastolic heart failure? Bratton’s
A. CCB + α-blocker
B. β-Blocker + diuretic
C. ACE inhibitor + α-blocker
D. CCB + ARB
E. ARB + CCB
Hypertension
93. Which of the following is definition of hypertension?
A. Systolic blood pressure ≥130 mm Hg, diastolic blood pressure ≥80;
B. Systolic blood pressure ≥130 mm Hg, diastolic blood pressure ≥90;
C. Systolic blood pressure ≥140 mm Hg, diastolic blood pressure ≥85;
D. Systolic blood pressure ≥140 mm Hg, diastolic blood pressure ≥90;
E. Systolic blood pressure ≥150 mm Hg, diastolic blood pressure ≥90;
95. Which of the following disease may cause a secondary arterial hypertension? P.129
A. Sleep apnea
B. Kidney vascular disease
C. Pheochromocytoma
D. All of the above
Ahmad Soboh 23
96. Which of the following disease may cause a secondary arterial hypertension? P.129
A. Cushing's syndrome
B. Primary aldosteronism
C. Chronic kidney disease
D. Thyroid or parathyroid disease;
E. All of the above
97. Which of the following disease may cause a secondary arterial hypertension
A. Primary aldosteronism
B. Chronic kidney disease
C. Cushing syndrome
D. Sleep apnea
E. All of the above
100. Which of the following is not considered to be routine test during primary
evaluation of patients with high blood pressure? P.128
A. Blood creatinine
B. Blood glucose
C. Urinalysis
D. BNP
101. Which of the following is not considered to be routine test during primary
evaluation of patients with high blood pressure?
A. Pottasium;
B. Blood glucose test
C. Creatinine:
D. Chest x-ray;
E. Lipid test;
Ahmad Soboh 24
102. Which of the following is not considered to be routine test during primary
evaluation of patients with high blood pressure?
A. Potassium
B. Urinalysis
C. Fasting lipid profile
D. INR
103. Which of the following is not considered to be routine test during primary
evaluation of patients with high blood pressure?
A. Blood glucose
B. Blood creatinine
C. Cholesterol
D. 24 hour urine test
104. Which of the following is not considered to be routine test during primary
evaluation of patients with high blood pressure ?
A. Blood creatinine;
B. Urinalysis:
C. Blood glucose;
D. Fasting lipid profile;
E. Prothrombin index;
105. Which of the following is not considered to be routine test during primary
evaluation of patients with high blood pressure?
A. Fasting glucose
B. Potassium
C. ECG;
D. lipid profile
E. ALT, AST
106. Which of the following is not considered to be routine test during primary
evaluation of patients with high blood pressure?
A. CBC and Urynalisis
B. Fasting glucose test
C. BNP
D. Potassium
E. Fasting lipid profile
107. All of the following interventions are important for reduction of high blood
pressure except P.130
A. Salt restriction
B. Prescribing aspirin
C. Exercise
D. Losing weight
E. DASH diet
Ahmad Soboh 25
108. A 39-year-old white man with essential hypertension presents for a routine health
maintenance visit. He has no complaints and reports compliance with his
hydrochlorothiazide. His pulse is 70/min, blood pressure is 145/92 mm Hg, and
respiratory rate is 16/min. His body mass index is 24 kg/m2. His physical examination
is within normal limits. For which condition is the patient at increased risk? First Aid – CK
A. End-stage renal disease
B. Hypercholesterolemia
C. Hypertrophic cardiomyopathy
D. Second-degree Mobitz I atrioventricular block
E. Type 2 diabetes mellitus
→ Hypertension is a risk factor for both chronic renal insufficiency and end-stage renal disease. Hypertension can directly
cause renal disease and accelerate the progression of underlying renal pathology. In addition, hypertension increases a
patient’s risk of premature cardiovascular disease, heart failure, stroke, and intracerebral hemorrhage.
109. A 53-year-old with type 2 diabetes mellitus is found to have a blood pressure
of 152/98. She has never had any ophthalmologic, cardiovascular, or renal
complications of diabetes or hypertension (HTN). Based on recent
recommendations of the JNC 7 (The Eighth Report of the Joint National
Committee on Prevention, Detection, Evaluation, and Treatment of High Blood
Pressure) which of the following is the currently recommended goal for blood
pressure control in this case?
A. Less than 160/90
B. Less than 145/95
C. Less than 140/90
D. Less than 130/80
E. Less than 120/70
→ Goals for blood pressure control and lipid levels are typically more stringent in the diabetic compared to the nondiabetic.
The goal blood pressure for diabetics and patients with renal disease is <130/80. Blood pressure goal for the standard patient is
<140/90. Both systolic and diastolic pressures should be below goal in order to achieve optimal blood pressure control.
110. A 28-year-old woman has hypertension that is difcult to control. She was
diagnosed at age 26. Since that time, she has been on increasing amounts of
medication. On physical examination, she appears to be without distress.
Blood pressure is 168/100 mmHg, and heart rate is 84 bpm. Cardiac
examination is unremarkable, without rubs, gallops, or murmurs. She has
good peripheral pulses and no edema. Her physical appearance does not
reveal any hirsutism, fat maldistribution, or abnormalities of genitalia.
Laboratory studies reveal a potassium of 2.8 mE/dL and a serum bicarbonate
of 32 mE/dL. Fasting blood glucose is 105 mg/dL
A. Primary hyperaldosteronism
B. Hypertrophic cardiomyopathy
C. Congenital adrenal hyperplasia
D. Cushing syndrome
E. Fibromuscular dysplasia
F. Pheochromocytoma
→ also known as Conn syndrome
Ahmad Soboh 26
112. Despite lifestyle changes, a 37-year-old patient of yours still has blood
pressures above goal. She has no other medical concerns and no abnormalities
on physical examination or initial laboratory evaluation. Which of the
following medications is best as an initial first-line monotherapy, according to
the Joint National Committee 7 (JNC 7)? Pre-Test
A. A thiazide diuretic
B. An ACE inhibitor
C. An angiotensin receptor blocker
D. A calcium channel blocker
E. A β-blocker
→JNC 7 recommended that low-dose diuretics are the most effective first-line treatment for preventing the
occurrence of cardiovascular morbidity and mortality.
113. You are seeing a 47-year-old hypertensive patient in your office. He is well-
controlled with hydrochlorothiazide and is seeing you for a routine evaluation.
His blood pressure at the visit is 118/76 mm Hg. Laboratory evaluation reveals
a normal creatinine but he does have A1c 7.2. Which of the following
interventions is indicated in this patient? Pre-Test
A. Work to achieve better blood pressure control through diet and exercise.
B. Add an angiotensin-converting enzyme (ACE) inhibitor
C. Commend him on his excellent control and make no changes
D. Add calcium channel-blocker
→ According to National Kidney Foundation guidelines, this patient has stage 1 renal failure. ACE inhibitors should
be added to his regimen to prevent the evolution of microalbuminuria to full blown proteinuria.
114. You are seeing a 43-year-old hypertensive patient in your office. He is well-controlled
with hydrochlorothiazide and is seeing you for a routine evaluation. His blood
pressure at the visit is 118/76 mm Hg. Laboratory evaluation reveals a normal
creatinine and a GFR greater than 90 mL/min, but he does have microalbuminuria.
Which of the following interventions is indicated in this patient? Pre-Test
A. Commend him on his excellent control and make no changes.
B. Work to achieve better blood pressure control through diet and exercise.
C. Increase his hydrochlorothiazide dose.
D. Add an angiotensin-converting enzyme (ACE) inhibitor.
E. Check a glycosolated hemoglobin level.
→ According to National Kidney Foundation guidelines, this patient has stage 1 renal failure. ACE inhibitors should
be added to his regimen to prevent the evolution of microalbuminuria to full blown proteinuria.
Ahmad Soboh 27
115. You have diagnosed a 35-year-old African-American man with hypertension. Lifestyle
modifications helped reduce his blood pressure, but he was still above goal. You chose to
start hydrochlorothiazide, 25 mg daily. This helped his blood pressure, but it is still 142/94
mm Hg. Which of the following is the best approach to take in this situation? Pre-Test
A. Increase his hydrochlorothiazide to 50 mg/d.
B. Change to a loop diuretic.
C. Change to an ACE inhibitor.
D. Change to a β-blocker.
E. Add an ACE inhibitor.
116. A 63-year-old woman has Type II diabetes mellitus, which is well-controlled. Her
physical examination is positive for peripheral neuropathy in the feet and
nonproliferative retinopathy. A urinalysis is positive for proteinuria. Which of the
following treatments is most likely to attenuate the course of renal disease?
A. calcium channel blockers
B. ACE inhibitors
C. hepatic hydroxymethyl glutaryl- coenzyme A (HMG-CoA) inhibitors
D. dietary carbohydrate restriction
117. 63 years old woman with DM type 2, which is small controlled. Her physical
examination is positive for peripheral neuropathy in the feet and non-proliferative
retinopathy. Urinalysis is positive for proteinuria. One of the following treatments
is positive for attenuate the course of renal disease:
A. Beta blockers
B. ACE inhibitors
C. HMG-CoA
D. Dietary carbohydrate restriction
E. Weight reduction
118. A 55-year-old man comes to your office after not being seen by a physician in more
than 10 years. He is found to be hypertensive, and his creatinine is found to be 2.3
mg/dL (H). Which medication is most likely to control his blood pressure and
decrease the likelihood of progression of his renal disease?
A. A thiazide diuretic
B. An ACE inhibitor
C. A calcium channel blocker
D. A β-blocker
E. An aldosterone antagonist
Ahmad Soboh 28
119. A 51-year-old white man recently relocated to the area and presents to your office as a
new patient. He denies having any history of medical problems. He made very infrequent
visits to a primary care provider where he previously lived. He is on no medicines and
denies having any significant family medical history. He is a current smoker with a 40
pack-year smoking history. His blood pressure is 170/95 mm Hg, and a fourth heart sound
is present. His physical examination is otherwise unremarkable. Which of the following
statements regarding treatment of this patient's hypertension is true? ACP
A. The most appropriate initial medical therapy for this patient is an alpha blocker
B. The most appropriate initial medical therapy for this patient is a thiazide diuretic
C. The most appropriate initial medical therapy for this patient is a thiazide
diuretic in combination with another antihypertensive agent that works via a
different blood pressure regulatory pathway
D. To have this patient stop smoking cigarettes would have little or no effect on the
control of his hypertension
120. A 38-year-old African-American man is evaluated for the new diagnosis of hypertension.
His workup has shown multiple elevated blood pressure readings in the past few months
but no evidence of any other medical conditions. You plan to initiate an antihypertensive
medication. Which of the following would be the best initial choice? PPT
A. calcium channel blocker
B. angiotensin receptor blocker
C. B-bloker
D. angiotensin converting enzyme inhibitor
→ Prevalence, severity of HTN increased in African Americans.
→ African Americans demonstrate somewhat reduced BP responses to monotherapy with BBs, ACEIs, or
ARBs compared to diuretics or CCBs.
121. A 64-year-old black man presents to your office for routine follow-up care. You
have treated him for many years for hypertension with a calcium channel blocker
and a thiazide diuretic. His hypertension has been moderately well controlled
with this regimen. He asks you whether having a home blood pressure monitor
would be useful for his care. Which of the following statements regarding
ambulatory blood pressure monitoring (ABPM) is true? ACP
A. ABPM is not useful in patients whose office blood pressure is normal and
who have hypertensive target organ injury
B. ABPM is not a useful tool in the evaluation of suspected autonomic
dysfunction in patients with orthostatic hypotension
C. Cross-sectional studies show that blood pressure averages obtained during
office visits correlate better with the presence of target-organ injury
(especially LVH) than those obtained with ABPM
D. ABPM is useful in establishing a diagnosis of white-coat hypertension
Ahmad Soboh 29
122. You have seen a 36-year-old man with elevated blood pressure. On one occasion, his
blood pressure was 163/90 mm Hg, and on a second occasion, his blood pressure
was 158/102 mm Hg. You have encouraged lifestyle modifications including weight
loss using exercise and dietary changes. Despite some modest weight loss, at his
current visit, his blood pressure is 166/92 mm Hg. Which of the following is the best
treatment strategy at this point? Pre-Test
A. Use a thiazide diuretic.
B. Use an ACE inhibitor.
C. Use an angiotensin receptor blocker.
D. Use a β-blocker.
E. Use a two-drug combination of medications.
→ The patient in this question has stage 2 hypertension (systolic blood pressure ≥160 mm Hg, or diastolic blood
pressure ≥90 mm Hg). Since lifestyle modifications have not helped, the next step is to institute drug therapy. JNC
7 guidelines state that in patients with stage 2 hypertension, two-drug combination therapy is indicated.
Diabetes
124. You are evaluating a 56 year-old obese woman who complains of fatigue. She
has polydipsia, polyuria, polyphagia. Which of the following laboratory reports
confirms the diagnosis of diabetes? P.151
A. A glucose reading, taken 2 hours after a 75-g glucose load, of 163 mg/dL
B. A random glucose reading of 221 mg/dL
C. fasting glucose measurement of 120mg/di
D. A1C 5.0
→ Symptomatic (polyuria, polydipsia, weight loss) or hyperglycemic crises and a random plasma glucose > or
equal 200 mg/dL
125. You are evaluating a 36-year-old obese woman who complains of fatigue. She
denies polydipsia, polyuria, polyphagia, or weight loss. Which of the following
laboratory reports confirms the diagnosis of diabetes?
A. A random glucose reading of 221 mg/dL
B. A random glucose reading of 221 mg/dL, and another, on a later date, of
208 mg/dL
C. A fasting glucose measurement of 128 mg/dL
D. A glucose reading, taken 2 hours after a 75-g glucose load, of 163 mg/dL
E. A fasting glucose of 114 mg/dL, and a reading of 184 mg/dL 2 hours after a
75-g glucose load
→ The diagnosis of diabetes may be made by two separate random glucose measurements more than 200 mg/dL with classic
signs of diabetes (polydipsia, polyuria, polyphagia, weight loss), a fasting glucose greater than 126 mg/dL, or a glucose
reading greater than 200 mg/dL 2 hours after a 75-g glucose load.
126. A 52-year-old obese man presents for follow-up of his hypertension. His blood
pressure is well controlled on a daily dose of hydrochlorothiazide. A finger-stick
blood sugar test done an hour after he ate lunch was 130 mg/ dL. Which of the
following test results would be diagnostic for diabetes mellitus?
A. A random plasma glucose of 220 mg/ dL and symptomatic polyuria
B. The nonfasting, finger-stick sugar of 130 mg/dL is diagnostic. No further
testing is needed
C. A fasting plasma glucose of 120 mg/ dl
128. Which of the following is correct about hyperosmolar hyperglycemic state HHS: P.150
A. Physical examination reveals more mild dehydration, rapid, deep breathing, fruity
smelling breath
B. Physical examination reveals more severe dehydration, mental status changes,
focal neurologic signs (hemiparesis, visual field deficits), seizure
C. The typical presentation is rapid onset (<24 hours), with symptoms of nausea,
vomiting, abdominal pain
D. Hyperosmolar hyperglycemic state- HHS can present with high levels of
hyperglycemia but less than 1,000 mg/dl, with kelosis or acidosis
E. Hyperosmolar hyperglycemic state (HHS) is more likely occur in type 1 diabetes
131. A 39-year-old G1P0 woman who is a new patient presents to the office at 10
weeks' gestation. She is known to have type 2 diabetes mellitus and currently
takes metformin. Her BMI of 34.7. This patient is at an increased risk for
developing which of the following? Case-File
A. Fetal malformations
B. Fetal macrosomia
C. Polyhydramnios
D. Shoulder dystocia
Ahmad Soboh 32
132. A 53-year-old woman with type 2 diabetes mellitus is found to have a blood pressure
of 152/98. She has never had any ophthalmologic, cardiovascular, or renal
complications of diabetes or hypertension (HTN). Based on recent recommendations
of the JNC7 (The Eighth Report of the Joint National Committee on Prevention,
Detection, Evaluation, and Treatment of High Blood Pressure) which of the following is
the currently recommended goal for blood pressure control in this case? Med
A. Less than 160/90
B. Less than 145/95
C. Less than 140/90
D. Less than 130/80
E. Less than 120/70
→ Goals for blood pressure control and lipid levels are typically more stringent in the diabetic compared to the
nondiabetic. The goal blood pressure for diabetics and patients with renal disease is <130/80. Blood pressure goal for the
standard patient is <140/90. Both systolic and diastolic pressures should be below goal in order to achieve optimal blood
pressure control.
133. A 34-year-old man was diagnosed with type 2 diabetes mellitus as a child and has
been presenting for annual examinations since he was in his early 20s. Which of the
following screening tests is indicated more than once a year
A. Microalbuminuria
B. Podiatry examination
C. Hemoglobin A1c
D. Ophthalmologic examination
E. Lipid profile
→ A hemoglobin A1c level should be checked every 3 months to assess the average serum glucose levels over that
time period. This is important in adjusting medications and to follow the disease progression.
134. In which of the following categories should women undergo routine screening
for gestational diabetes? Harrison’s
A. Age greater than 25 years
B. Body mass index greater than 25 kg/m2
C. Family history of diabetes mellitus in a first-degree relative
D. African American
E. All of the above
Diabetes Drugs
136. A 53-year-old woman's A1C is 7.6%. She has attempted to lose weight and to
exercise with no improvement in her hemoglobin A1C, and drug therapy is now
recommended. She has mild systemic hypertension that is well controlled and her
LDL is 120mg/dL. Which of the following is the most appropriate first-line therapy?
A. Acarbose
B. Sitagliptin
C. Metformin
D. Metformin and statin
A1c → she need to management by Metformin
LDL is 120 mg/dL → she need to management the LDL level by Statin
137. Patient with newly established diagnosis of type 2 diabetes mellitus. Along with diet
and exercise, which of the following is the most appropriate initial management?
A. Single daily injection of glargine insulin
B. Mealtime injections of short-acting insulin
C. Oral metformin
138. A 49-year-old woman presents to her physician's office with a long-standing history
of polydipsia, polyuria, central obesity, and hyperlipidemia. She is currently taking
metformin, a sulfonylurea, and an angiotensin-converting enzyme (ACE) inhibitor.
ACE inhibitors are most beneficial in preventing or slowing the progression of which
of the following diabetic complications? First Aid – CK
A. Diabetic ketoacidosis
B. Diabetic nephropathy
C. Diabetic neuropathy
D. Diabetic retinopathy
E. Peripheral vascular disease
→ ACE inhibitors such as captopril have been shown to decrease blood pressure and prevent
and slow the progression of diabetic nephropathy in patients with DM. It is believed that ACE
inhibitors play a renoprotective role by reducing glomerular filtration rate and reducing
macroproteinuria.
Thyroid
139. A 26-year-old woman presents with weight gain, lethargy, dry skin,
sweatiness, cold intolerance, and thinning hair. You suspect hypothyroidism
and order the appropriate laboratory tests. Her TSH is high, and her free T 3
and free T 4 are both low. Which of the following is the most likely diagnosis?
A. Primary hypothyroidism
B. Secondary hypothyroidism
C. Iodine deficiency
D. Thyroid hormone resistance
E. Subclinical hypothyroidism
Ahmad Soboh 34
140. You have been following a 45-year-old man for hypothyroidism for the past 2
years without significant complaints. He was prescribed 100 g of levothyroxine 1
year ago, when the diagnosis was made, and his TSH fell to normal levels at the 3-
month and 6-month visits. He returns for a routine annual recheck and manifests a
normal blood pressure and apical heart rate of 80, regular. As he has a family
history of adenomatous colon polyps, you arrange for colorectal cancer screening
annually this decade. Then you draw routine fasting lipids because he has gained 5
lb (2.26 kg) during the past year and also draw in the same specimen serum TSH.
The test results return normal except for TSH, whose level is 6.2 IU/mL (normal is
0.4 to 4.8). Each of the following may constitute explanations for this finding,
except which factor? NMS
A. Failure of patient to comply with the prescribed dosage of levothyroxine.
B. Inadequate dosage of levothyroxine prescribed.
C. The prescribed levothyroxine is malabsorbed.
D. The patient has secondary hypothyroidism.
E. There may be a comorbid autoimmune condition.
142. You are caring for a 35-year-old man who is complaining of fatigue and an
inability to gain weight. Laboratory evaluation reveals a TSH of 6.0 mIU/L (H)
but a normal free T 4. Which of the following is the best next step?
A. Test for antithyroid peroxidase.
B. Test for thyroid autoantibodies.
C. Treat with levothyroxine.
D. Treat with levothyroxine and T 3.
E. Monitor at yearly intervals.
144. Which of the following test results supports the diagnosis of Graves' disease?
A. Decreased TSH
B. Increased TSH
C. Decreased thyroxine (T4) levels
D. Decreased triiodothyronine (T3) levels
E. None of the above
148. A 28 year-old woman is noted to have had 10-Ib unintended weight gain hair
loss, dry skin and fatigue. She is diagnosed with probable hypothyroidism.
Which of the following laboratory test results is most consistent with
hypothyroidism?
A. Normal TSH and elevated T /T 3 levels
B. Elevated TSH levels and normal T /T 3
C. Elevated TSH levels and low T /T 3
D. Low TSH and elevated T /T 3 levels
Ahmad Soboh 36
149. A 30-year-old female complains of fatigue, constipation, and weight gain. There is no
prior history of neck surgery or radiation. Her voice is hoarse and her skin is dry.
Serum TSH is elevated and T4 is low. The most likely cause of these findings is:
A. Autoimmune disease
B. Postablative hypothyroidism
C. Pituitary hypofunction
D. Thyroid carcinoma
150. A 35-year-old woman presents to her primary care physician because of 2 weeks of
neck pain. The pain is constant and sharp (rated at 10 of 10) and is felt in the anterior
portion of her neck. She also notes several weeks of loose stools and fatigue. Past
medical history is significant for a viral upper respiratory infection about 1 month
ago. She has a temperature of 37.9°C (100.2°F), heart rate of 96/min, and blood
pressure of 136/82 mm Hg. On neck examination there is diffuse enlargement of the
thyroid and it is exquisitely tender to even mild palpation. Her oropharynx.
Laboratory studies are sent and reveal a white blood cell count of 14,100 cells/ML
with a normal differential, erythrocyte sedimentation rate (ESR) of 53 mm/h. Thyroid
antibodies are negative. What is the most likely
A. Autoimmune hypothyroidism
B. Drug-induced thyroiditis
C. Hashimoto’s thyroiditis
D. Subacute thyroiditis;
E. All of the following
151. A 40-year-old woman presents to the outpatient clinic for a routine visit. On
physical examination a 1-cm nodule is palpated in her thyroid. Her physical
examination is otherwise unremarkable. Her heart rate is 70/min and regular,
blood pressure is 126/82 mm Hg, and temperature is 36.7oC (98.0oF). Which of
the following is a poor prognostic indicator for the thyroid nodule?
A. Female gender
B. Hoarseness
C. Patient age of 56 years
D. Slow growth of nodule
E. Tender nodule
→ Hoarseness generally implies vocal cord impairment due to tumor involvement of the recurrent laryngeal nerve. This suggests a
malignant tumor that has extended beyond the thyroid and invaded local structures. This is a poor prognostic indicator. Local
invasion is particularly common with papillary carcinoma.
Ahmad Soboh 37
152. A 72-year-old man with atrial fibrillation presents with complaints of fatigue
and feeling cold. He also notes constipation and dry skin. His daughter states
he has seemed more forgetful over the past several months. His temperature
is 37.3°C (99.1°F), heart rate is 48/min, and blood pressure is 130/82 mm Hg.
Cardiac examination shows bradycardia but normal rhythm, and normal S1
and S2 with no murmurs; the lungs are clear to auscultation bilaterally and the
abdomen is soft and nontender. The patient’s extremities are cool and puffy
with dry, coarse skin. Laboratory studies show a thyroid-stimulating hormone
level of 32 μU/L, free thyroxine of 0.3 ng/dL, and total triiodothyronine of 30
ng/dL. What medication is the patient likely taking for his atrial fi brillation?
A. Amiodarone
B. Flecainide
C. Lithium
D. Methimazole
E. Sotalol
→ The patient is presenting with signs and symptoms consistent with hypothyroidism which include fatigue,
weakness, cold intolerance, dry skin, constipation, bradycardia, coarse hair and skin, and puffy, cool extremities.
154. The mother of a 9-month-old infant brings him in for irritability. The child has
been fussy and has not been sleeping well for 2 days. His highest temperature
has been 100°F, and he has had a clear runny nose and cough. On
examination, the child is crying and irritable. Which physical examination
finding, by itself, is insufficient to diagnose acute otitis media? Pre-Test
A. Opaque tympanic membrane
B. Bulging tympanic membrane
C. Impaired tympanic membrane mobility
D. Erythematous tympanic membrane
E. Purulent discharge in the ear canal
155. A 60 year old female patient has been complaining of left earache of 3 months
duration. One month ago a swelling appeared in the left side of the neck that
progressively increased in size. 2 days ago cot complained of change of her
voice together with dysphagia. On indirect laryngoscopy there was froth in the
region behind the larynx. What is the most possible diagnosis
A. Left pyriform fossa malignancy with lymph node metastasis
B. Otitis media complicated with limphadenopathy
C. Peritonsillar abscess
→ Left pyriform fossa malignancy (left earache, change in voice and dysphagia, froth behind the larynx) with lymph
node metastasis (swelling on the left side of the neck progressively increasing in size)
Ahmad Soboh 38
157. A 25 year old male presented to the ENT emergency room with severe right
side throat pain, inability to swallow, accompanied by right earache of 2 days
duration. The patient was unable to open his mouth and was feverish 40 C. On
examination there was a tender swelling at the angle of the mandible The
patient gave a history of sore throat and fever 39 C during the last week. What
is the most possible diagnosis in this case?
A. Right pyriform fossa malignancy wroth lymph node metastasis
B. Peritonsillar abscess
C. Acute otitis media
D. Acute tonzillitis
→ Acute tonsillitis (history of sore throat and fever) complicated by right peritonsillar abscess – quinzy (right sided throat
pain, inability to swallow and to open the mouth, fever 40 C)
• Right sided throat pain: due to the collection of pus in the peritonsillar pain that causes immense throbbing pain
• Inability to swallow: marked dysphagia accompanying the quinzy that may lead to drooling of saliva from the
mouth
• Right earache: refered pain along the glossopharyngeal nerve (Jackobsen's nerve)
• Unable to open the mouth: trismus caused by spasm of the medial pterygoid muscle present lateral
• to the peritonsillar abscess
• Tender swelling at the angle of the mandible: inflammed jugulodigastric lymphadenitis
160. A 21-year-old woman presents with a complaint of sore throat. She was in her
usual state of health until 3 days ago, when she developed a nonproductive cough,
nasal drainage, ear pain, and a sore throat. She denies having shortness of breath,
sputum production, fever, rash, joint pains, or gastrointestinal symptoms. She also
denies having been in contact with sick persons. For the past 2 years, she has been
in a sexual relationship with a single sexual partner. On physical examination, the
patient is found to have erythema of the posterior pharynx and nasal turbinates.
Small, bilateral, serous, middle-ear fluid collections are noted. Lung examination is
normal. The patient is afebrile. She requests antibiotics, stating that she always
improves much more quickly with this therapy. Which of the following statements
regarding pharyngitis is true? ACP
A. Group A streptococci are the most common cause of pharyngitis
B. Four clinical criteria have been proposed as suggestive of group A
streptococcal pharyngitis: tonsillar exudates, tender anterior
adenopathy, absence of cough, and history of fever
C. Office-based rapid diagnostic tests for group A streptococcal pharyngitis
have a sensitivity of nearly 100%
D. Pneumococci and staphylococci are emerging causes of pharyngitis
Gastrointestinal Problems
Abdominal Pain
162. Which of the following causes of Abdominal Pain Based on Location is correct: P.219
A. Diffuse: generalized peritonitis from perforation or rupture of an organ
B. Right lower quadrant (RLQ): appendix
C. Right upper quadrant (RUQ): gallbladder
D. All of the above
Ahmad Soboh 40
163. Which of the following causes of Abdominal Pain Based on Location and
Quality is correct:
A. Diffuse: generalized peritonitis from perforation or rupture of an organ
B. Right upper quadrant (RUQ): gallbladder
C. Left lower quadrant (LLO): sigmoid colon
D. Right lower quadrant (RLQ): appendix
E. All of the above
165. A 53-year-old obese woman presents to your office complaining of the recent
onset of abdominal pain. She describes pain that starts in the mid epigastric
region, radiating to the back. It is associated with nausea and vomiting. Past
history of alcohol use is negative. What is the most likely diagnosis?
A. Acute appendicitis
B. Pancreatitis
C. Peptic ulcer disease
D. Gastroesophageal reflux disease (GERD)
166. A 42-year-old woman presents to your office complaining of the recent onset of
abdominal pain. She describes pain that starts in the midepigastric region, radiating to
the back. It is associated with nausea and vomiting. What is the most likely diagnosis?
A. Acute appendicitis
B. Pancreatitis
C. Gallbladder disease
D. Esophageal spasm
E. Gastroesophageal reflux disease (GERD)
→ Pancreatitis generally settles in the midepigastric region with radiation to the back and is associated with nausea and vomiting.
169. A 58-year-old woman presents to her internist with a 2-day history of low-grade
fever and RUQ abdominal pain. She reports nausea but denies vomiting or diarrhea.
On physical examination, she is pale and jaundiced. Her temperature is 38.1°C. She
has RUQ tenderness without rigidity or rebound tenderness. Bowel sounds are
normal. Her white blood cell count is 16,000/μL and urinalysis shows no red or white
blood cells. Which of the following is the most likely diagnosis?
A. Right-sided pyelonephritis
B. Right lower lobe pneumonia
C. High retrocecal appendicitis
D. Cholecystitis → Cholangitis
E. Gastritis
→ All of the given diagnoses can cause epigastric or RUQ abdominal pain, but jaundice makes cholangitis the most likely diagnosis.
Fever, RUQ abdominal pain, and jaundice are called Charcot triad and are highly suggestive of cholangitis.
173. An 80-year-old man presents with mild, crampy, bilateral lower quadrant
pain, decreased appetite, and low-grade fever for about 48 hours. Which of
the following is the most likely diagnosis? Pre-Test
A. Small-bowel obstruction
B. Appendicitis
C. Constipation
D. Irritable-bowel syndrome (IBS)
E. Pancreatitis
174. 50 year-old women presents with mild, crampy, right lower quadrant pain,
decreased appetite, and low-grade fever for about 48 hours. Which of the
following is the most likely diagnosis
A. Small-bowel obstruction
B. Appendicitis
C. Constipation
D. Irritable-bowel syndrome (IBS)
E. Pancreatitis
178. A 55-year-old female enters an urgent care center with a complaint of acute
abdominal pain. She relates two attacks of less severe abdominal pain within
the past 3 months. The pain radiates through to the back opposite the
epigastric area and seems to be relieved by leaning forward. Each of the
following is true regarding this case except NMS
A. The most sensitive diagnostic test is the serum amylase.
B. Lipase elevation begins within 4 to 8 hours and remains elevated 8 to 14 days.
C. The vast majority of attacks are mild and not fatal.
D. The severity of disease is directly proportionate to the size of the amylase
elevation.
E. This patient must be treated to prevent delirium tremens while managing the
underlying condition.
179. A 72-year-old woman is in the surgical intensive care unit after a carotid
endarterectomy. She has been taking clindamyein and ciprofl oxacin for the past
13 days. On postoperative day 2 the patient is febrile and tachycardic with a high
WBC count and a low RBC count. She is also noted to be dehydrated and
hypotensive. On physical examination she is distended and has abdominal
tenderness with rebound andguarding. Stool is sent for Gram stain and analysis
for fecal leukocytes, fecal occult blood, and Clostridium diffi cile toxin. Which of
the following is most likely present in the stool sample? CK-USMLE
A. Gram-positive cocci
B. Gram-positive cocci
C. Spores and hyphae
D. Clostridium diffi cile toxin
E. Gram-negative rods
181. A 63-year-old man with diabetes is called by his primary care physician because of
abnormal liver function test results, as follows: Aspartate aminotransferase: 85 U/L,
Alanine aminotransferase: 102 U/L, Alkaline phosphatase: 180 U/L, Total bilirubin: 1.9
mg/dL On physical examination his liver is enlarged. His skin has a slightly yellow hue,
especially on his face. The review of symptoms is signifi cant for some weight loss,
weakness, arthritis in his hands, and inability to achieve an erection. What test would
generate the most likely diagnosis
A. Blood smear
B. CT scan of the abdomen
C. Endoscopic retrograde cholangiopancreatography
D. Fasting transferrin saturation levels
E. Liver biopsy
Dyspepsia
182. A 41-year-old man presents for evaluation of upper Gl discomfort present over the
last 2 months. He says that he has a "full" sensation in the epigastric region. He
denies blood in his stool, denies vomiting, and has had no dysphagia. He has lost 10
lb in the last few weeks unintentionally, which he attributes to not eating. His
mother has hemorrhoids, and no family member has ever had colon cancer. Which of
the following is the most appropriate next step in workup of this patient? Case-File
(Dyspepsia)
A. Colonoscopy
B. PPI therapy for 8 weeks
C. Fecal occult blood test
D. Upper endoscopy
→ This patient presents with the alarm symptom of weight loss. He should be referred for early endoscopy.
183. Which of the following H pylori eradication therapy regimen is correct: P.249 (Dyspe)
A. Bismuth subsalicylate and ornidazole
B. Metronidazole and tetracycline
C. Ciprofloxacin, tetracycline and PPI;
D. PPI (proton pump Inhibitor), clarithromycin and amoxicillin
E. H2 receptor antagonist, Bismuth subsalicylate and ornidazole;
184. 60-year-old man comes with persistent epigastric pain and nausea. Pain began
after taking NSAIDs for headache. He took PPI for 2 weeks with mild relief. After
stopping PPI pain began again. He has loss of appetite and occasionally nausea. He
is a smoker, without other medical problems. Physical examination shows
tenderness in epigastric area. Upper endoscopy reveals gastric erythema and a
mass in the body of the stomach. Biopsy is competent with low-grade lymphoma
from the mucosa associated lymphoid tissue (MALT). Which of the following is most
likely involved in the pathogenesis of this condition? P.248 (Dyspepsia)
A. Pernicious anemia
B. Cigarette smoking
C. Salt-preserved foods
D. Helicobacter pylori infection
Ahmad Soboh 45
185. A 62-year-old man presents with dark tarry stools and light-headedness. Upper
endoscopy finds an ulcerating lesion in his stomach and biopsies confirm gastric
cancer. Which of the following is a risk factor for carcinoma of the stomach?
A. Helicobacter pylori infection
B. High socioeconomic status
C. High protein diet
D. High alcohol consumption
E. high fat diet
→ H. pylori is another important risk factor, since it causes gastritis that eventually leads to chronic atrophic
gastritis, metaplasia, dysplasia, and then carcinoma.
186. Which of the following is the most important, productive, and practical
reason to offer or refer for upper endoscopy in any patient over 45 years with
persistent dyspepsia? NMS
A. To diagnose gastric peptic ulcer
B. To diagnose gastroesophageal reflux
C. To diagnose early stomach cancer
D. To diagnose peptic duodenal ulcer disease
E. To diagnose atrophic gastritis
→ Perhaps of the greatest surprise, gastric cancer can be a screenable disease.
187. What may be the possible diagnosis for 3-week-old boy with 2 days of
projectile nonbilious vomiting and constant feeding Case-File
A. Malrotation with intermittent volvulus
B. Intussusception
C. Esophageal foreign body
D. Pyloric stenosis
E. Volvulus
→ The young age and presence of projectile, nonbilious vomiting after feeding are the keys to this
diagnosis. The diagnosis of pyloric stenosis is much more common in males than females.
188. A new mother brings her infant to see you to discuss his vomiting. He is 4 weeks
old and is exclusively breast-fed. He vomits with every meal. On examination, his
abdomen is distended with normal bowel sounds, and he appears dehydrated. He
has lost 4 oz since his visit with you 2 weeks ago. Which of the following is the most
likely diagnosis?
A. Allergy to breast milk
B. GERD
C. Pyloric stenosis
D. Intussusception
E. Small-bowel obstruction
Ahmad Soboh 46
Liver
189. You are caring for a 12-year-old boy with psychiatric and neurologic
abnormalities. Laboratory evaluation show high serum and urine copper
levels. Eye examination reveals Kayser Fleischer rings. Which of the following
is the most likely cause of his condition?
A. Wilson's disease
B. Hemochromatosis
C. Autoimmune hepatitis
D. Viral hepatitis
190. A 15-year-old girl presents with malaise, fatigue, depression, and clinical evidence of
hepatitis in the absence of viral causation. An older sister is awaiting liver
transplantation. Which of the following blood studies may be most specifically helpful
in establishing the diagnosis? NMS
A. ANA levels
B. Anti-SMA levels
C. Complete blood count
D. Ceruloplasmin level
E. Serum iron, total iron-binding capacity, and ferritin levels
191. You are evaluating a 65-year-old man who complains about arthritis, heart disease,
erectile dysfunction. Laboratory evaluation reveals high blood iron transferrin
saturation. Which of the following is the most likely cause of her problem? P.229
A. Wilson disease
B. G6PD deficiency
C. Gilbert disease
D. Hemochromatosis
→ Hemochromatosis, a common genetic disorder of iron absorption that leads to iron overload, may initially
present with isolated transaminase elevation and, if left untreated, may lead to end-stage liver disease. Presence
of arthritis, erectile dysfunction, and heart failure in the patient with abnormal liver tests should prompt
evaluation for hemochromatosis.
192. A 45-year-old woman presents with fatigue, anorexia, brown urine, and malaise. She
denies alcohol intake entirely, illicit drug use, and history of treatment with blood
products, and she is on no medications that may cause elevation of hepatocellular
enzymes. Examination reveals scleral icterus and mildly tender hepatomegaly. Both AST
and ALT levels are elevated to four times normal, and smooth muscle antibody (SMA)
and antinuclear antibody (ANA) levels are positive. She has a history of Hashimoto
thyroiditis, but no history suggestive of inflammatory bowel disease. Alkaline
phosphatase and gamma-glutamyl transpeptidase are both elevated to twice normal
limits. A hepatitis profile yields no evidence of hepatitides A, B, C, or E. Which of the
following may be the best initial presumptive diagnosis?
A. PSC
B. PBC
C. Viral hepatitis
D. Choledocholithiasis
E. Autoimmune hepatitis
Ahmad Soboh 47
193. Which of the following statements is true about chronic hepatitis B? P.233 (Liver)
A. Screening for HBV includes measure HBV DNA levels
B. HBV is defined as chronic if an acute HBV infection persists more than 1 months;
C. Chronic HBV don't increases the risk of cirrhosis and HCC;
D. Serum HBsAg is an indication for HBV infection, whereas HBsAb is a marker of
immunity or vaccination to HBV;
E. All of the above
194. A 7 month male infant is brought to his pediatrician's office for a well-child visit.
He has a temperature of 37.2°C (99°F) and clear nasal drainage. Physical
examination is otherwise unremarkable. The infant is able to lift his head when
prone, track past the midline, and coo. His immunization record indicates that he
received is first hepatitis B vaccination at birth. Assuming this child is developing
normally, which vaccines should this infant receive today? First Aid – CK (Infect)
A. Hepatitis B, diphtheria-tetanus-pertussis, Haemophilus influenzae type
b, inactivated poliomyelitis, and pneumococcal conjugate
B. Hepatitis B and diphtheria-tetanus-pertussis only
C. None; schedule a return visit when the child's nasal congestion subsides,
and administer the Immunizations
D. Hepatitis B, diphtheria-tetanus-pertussis, and inactivated poliomyelitis
E. Hepatitis B only
195. A 19-years- old sexually active homosexual man asks you about his risks for
viral hepatitis. He is currently asymptomatic and unsure of his immune status.
Which of the following should you recommend? Pre-Test
A. Vaccination against hepatitis A only
B. Vaccination against hepatitis B only
C. Vaccinations against both hepatitis A and B
D. Vaccination against hepatitis A, B and C
196. Which of the following is the best indicator of a high degree of hepatitis B
virus (HBV) replication, hence marker of infectivity?
A. HBsAg
B. HBcAg
C. HBeAg
D. Anti-HBs
E. Anti-HBc
197. You check serologies on a patient exposed to hepatitis B. His serologies are:
HBsAg-Negative, HBeAg-Negative, Anti-HBs- Positive, Anti-Hb core IgG-
positive. Which of the following terms best describes his disease status
A. Acute infection, early phase
B. Acute infection, recovery phase
C. Previous exposure with immunity
D. Vaccination
Ahmad Soboh 48
198. You check serologies on a patient exposed to hepatitis B. His serologies are shown
below: • HBsAg: Negative • HBeAg: Negative • Anti-HBs: Positive • Anti-HBe:
Negative Which of the following terms best describes his disease status?
A. Acute infection, early phase
B. Acute infection, window phase
C. Acute infection, recovery phase
D. Previous exposure with immunity
E. Vaccination
199. Which of the following forms of hepatitis does NOT have a chronic state?
A. Hepatitis A
B. Hepatitis B
C. Hepatitis C
D. Hepatitis D
200. Which of the following is the most easily passed by a blood donor?
A. Hepatitis A
B. Hepatitis B
C. Hepatitis C
D. Hepatitis D
201. The Center for Disease Control and Prevention recommends that the following high-risk
individuals should be tested for HCV infection. Which of following are a high risk: P.235
A. Intravenous drug users
B. People with HIV infection
C. Individuals with unexplained abnormal liver function tests
D. All of the above
E. Non of the above
202. Which of the following is the next most common cause of blood transfusion-related
hepatitis after hepatitis B? Lange
A. hepatitis A
B. Epstein-Barr hepatitis
C. hepatitis C
D. hepatitis D
E. HIV hepatitis
203. A 26-year-old woman presents to your clinic and is interested in getting pregnant. She
seeks your advice regarding vaccines she should obtain, and in particular asks about
the hepatitis B vaccine. She works as a receptionist for a local business, denies alcohol
or illicit drug use, and is in a monogamous relationship. Which of the following is true
regarding hepatitis B vaccination? Harrison’s (Liver)
A. Hepatitis B vaccine consists of two IM doses 1 month apart.
B. Only patients with defined risk factors need to be vaccinated.
C. Pregnancy is not a contraindication to the hepatitis B vaccine.
D. This patient’s hepatitis serologies should be checked before vaccination.
E. Vaccination should not be administered to children under 2 years old.
Ahmad Soboh 49
204. A patient that you follow has recently started volunteering at a drug treatment hospital
and requires hepatitis B vaccination. You find that he is hepatitis B surface antibody
positive. Which of the following would be the best guideline to follow in this case?
A. No vaccination is necessary based on his laboratory evaluation.
B. Administer one dose of hepatitis B vaccine.
C. Administer two doses of hepatitis B vaccine, at least 1 month apart.
D. Administer two doses of hepatitis B vaccine, at least 6 months apart.
E. Administer three doses of hepatitis B vaccine at the appropriate time interval.
205. A patient requires hepatitis B vaccination. You find that he is hepatitis B surface antibody
positive. Which of the following would be the best guideline to follow in this case
A. No vaccination is necessary based on his laboratory evaluation.
B. Administer one dose of hepatitis B vaccine.
C. Administer two doses of hepatitis B vaccine, at least 1 month apart.
D. Administer two doses of hepatitis B vaccine, at least 6 months apart.
206. Which of the following immunizations should be given in the newborn period? NMS
A. Diphtheria, tetanus, pertussis (DTaP), first dose
B. Hemophilus influenzae type B (Hib), first dose
C. Inactivated polio vaccine (IPV), first dose
D. Pneumococcal vaccine (PCV), first dose
E. Hepatitis B (HepB), first dose
Lower intestinal
207. A 67 year old man comes to the office with 3 month history of fatigue and dyspnea
on exertion. She does not take medicines and did not have cardiovascular disorders.
Body temperature is normal, BP 160/80mm, pulse 110, respirations-22. Auscultation
-2/6 mid-systolic murmur in the second right intercostal space with normal S2
splitting. No peripheral edema is noted. Rectal examination shows guaiac negative
stools. ECG-sinus tachycardia. According to laboratory blood tests-Iron deficiency
anemia. Which of the following is the most appropriate next step?
A. Colonoscopy
B. Bone marrow biopsy
C. Echocardiography
D. Colonoscopy and endoscopy
→ Both colonoscopy and upper endoscopy should be considered in patients with iron deficiency anemia, because
it is often unclear whether a finding on one endoscopic exam represents the true cause of occult bleeding.
210. An 11-month-old male infant with intermittent bouts of crying and nonbilious
vomiting, with a history of Meckel diverticulum. A small, elongated mass is felt on
right side of his abdomen. What is the most possible diagnosis for this baby? P.253
Case-File
211. A 30-year -old man comes to the office due to low back pain and stiffness for 2
months. His pain is worse in the morning and improves with activity. The
patient has also had intermittent diarrhea and lower abdominal pain for the
past 3 months. He tried ibuprofen, which helped his back pain, but made his
diarrhea worse. he has not travelled abroad. He has no other medical
problems. Physical examination shows limited spine flexion and tenderness in
the lower back. lab, studies show anemia and thrombocytosis. Stool cultures
are negative. Which of the following is the most likely cause of this patient's
symptoms? U-world (L.Intest)
A. Celiac disease
B. Inflammatory bowel disease
C. Paraneoplastic syndrome
D. Reactive arthritis from diarrhea
E. Rheumatoid arthritis
F. Spinal osteomyelitis
→ Inflammatory bowel disease is frequently complicated by arthritis, which occurs in up to 45% of patients and can involve
axial or peripheral joints. Treatment with nonsteroidal anti-inflammatory drugs may exacerbate the underlying disease.
Ahmad Soboh 51
212. A 25-year-old woman comes to your office worried that she might have IBS, which
she heard about on the news. She reports abdominal pain and diarrhea for 3 months.
She also reports observing blood in her stool several times. Her physical examination
is normal except for a hemoccult-positive test after a rectal examination. While
looking over her records you notice that she has lost 20 lb since she last saw you 3
months ago. Which of the following is an appropriate next step? Case-File
A. Refer her for cognitive behavioral therapy.
B. Offer her symptomatic relief with loperamide.
C. Recommend that she take fiber for better bowel regulation.
D. Obtain colonoscopy.
→ This patient presents with alarm signs of blood in the stool and weight loss. Although psychiatric problems or
irritable bowel syndrome are possible, more serious conditions should be evaluated and ruled out. A CBC, erythrocyte
sedimentation rate, and colonoscopy or radiological study assessing for inflammatory bowel disease would be prudent.
213. A 40-year-old man contracts a bacterial gastroenteritis associated with fever, severe
abdominal pain, and profuse diarrhea. The etiologic agent is never determined. Which of
the following statements accurately characterizes the complications that may ensue in
this patient? ACP
A. As many as 40% of patients with Guillain-Barré syndrome had recent
Shigella infection
B. The arthritis associated with Campylobacter infection results from
bacteremic spread of infection to joints
C. Antibiotic treatment of infection caused by enterohemorrhagic E. coli (EHEC)
may increase the risk of developing hemolytic-uremic syndrome (HUS)
D. HUS most commonly results from infection with Shigella
E. The development of erythema nodosum suggests infection with Salmonella
214. A 22 year old man comes in the office with a 5 week history of diarrhea. There is no
blood in the stool. He recently returned from Africa, used unboiled water and had
had foul smelling stool, abdominal cramps and bloating. Antibiotic treatment did not
relieved his symptoms. His medical and family history is nor remarkable. He does not
take medicines, illicit drugs and is not a smoker. Body temperature and BP are
normal, pulse-86. Physical examination shows soft, non tender abdomen, with
increased bowel sounds. Which of the following is the best next step? U-world
A. Ciprofloxacin for 10 days
B. Colonoscopy with biopsy
C. CT scan of the abdomen
D. Stool antigen testing for Giardia
Giardiasis is characterized by persistent diarrhea and abdominal cramps, and is most commonly transmitted via water in rural
areas and developing countries. The preferred diagnostic test is a stool antigen assay. Metronidazole is the first-line treatment
215. Which of the following parasite is capable of causing intestinal malabsorption? NMS
A. Enterobius vermicularis
B. Giardia lamblia
C. Ancylostoma duodenale
D. Necator americanus
E. Entamoeba histolytica
Ahmad Soboh 52
216. A 31 year old man has acute diarrhea after he arrived from south America.
The most effective treatment of his diarrhea is: PPT: Small Int
A. Trimethoprim-sulfamethoxazole (TMP-SMX)
B. Doxycycline
C. Metronidazole
D. Ciprofloxacin
217. The most effective drug for the treatment of traveler's diarrhea is Bratton’s
A. Tetracycline
B. Trimethoprim-sulfamethoxazole (TMP-SMX)
C. Doxycycline
D. Metronidazole
E. Ciprofloxacin
Acute Respiratory
219. A 34-year-old man seeks evaluation of a cough. His illness began with a sore throat
and nasal congestion 5 days ago. He subsequently developed a cough productive of
green sputum and a burning sensation in the retrosternal chest that occurs with
coughing spells. He has felt cold at times but denies shaking chills, shortness of
breath, and hemoptysis. There is a history of seasonal allergies, but he takes no
medications and has no known drug allergies. He is a nonsmoker who typically jogs 3
miles, four times weekly. Pulse oximetry reveals a normal resting oxygen saturation;
the patient's temperature is 100.2° F (37.9° C). The chest is clear to percussion, with
audible expiratory wheezes. After taking a deep breath, the patient coughs,
producing green sputum. A Gram stain of the specimen reveals polymorphonuclear
and mononuclear cells without microorganisms. A chest x-ray is normal. Which of
the following would be the most appropriate choice for antimicrobial treatment of
this patient's condition? ACP
A. No antimicrobial treatment
B. Amoxicillin-clavulanate, 875 mg p.o., b.i.d., for 10 days
C. Trimethoprim-sulfamethoxazole, 1 double-strength tablet p.o., b.i.d., for 10 days
D. Azithromycin, 500 mg p.o. once, then 250 mg p.o., q.d., for 4 days
E. Levofloxacin, 250 mg p.o., q.d., for 10 days
→ This otherwise healthy man who is without underlying lung disease has acute bronchitis, an illness caused
predominantly by respiratory viruses. Up to 85% of patients diagnosed with acute bronchitis in the United States receive
antimicrobial therapy. This practice has likely contributed to the rapid emergence of drug-resistant strains of bacteria.
Ahmad Soboh 53
220. A 56-year-old patient who has smoked two packs of cigarettes a day for 40 years
presents to your office for a second opinion. His previous physician recently
diagnosed him as having chronic bronchitis. The patient reports that no work-up was
done, "not even a chest x-ray." The patient asks, "How can my doctor know I have
chronic bronchitis without doing any sort of examination or workup?" Which of the
following is the most appropriate response to this patient's question?
A. "You are right; at least a chest x-ray is needed to make this diagnosis."
B. "You are right; you need some blood work, lung function tests, and a chest
x-ray before this diagnosis can be made."
C. "Given your history of smoking, you must have chronic bronchitis."
D. "The diagnosis is made on the basis of history alone; given your smoking history,
if you have a cough with sputum production for most days for at least 3 months
each year for 2 or more years, chronic bronchitis is the correct diagnosis."
225. According to the recommendations of the Center for Disease Control and
Prevention, all of the following should receive the influenza vaccine except: P.633
A. All children between the ages of 6 months and 18 years
B. All infants younger than 6 months of age
C. Health care personnel
D. Pregnant individuals
Ahmad Soboh 54
226. You are caring for a family and find that the mother would like her children
vaccinated against influenza. Her children are aged 5 months, 22 months.
Which of the following represents current immunization recommendations for
influenza? Pre-Test
A. None of her children should be vaccinated.
B. The 5-month-old should be vaccinated.
C. The 22-month-old should be vaccinated.
D. Both should be vaccinated.
→ Influenza is a highly contagious viral infection. Vaccination is between 30% and 90% effective in preventing
influenza or complications from influenza. Influenza vaccination is recommended annually for children aged 6
months and older. The minimum age for vaccination with the trivalent inactivated influenza vaccine (TIV) is 6
months, but the minimum age for the live, attenuated influenza vaccine (LAIV) is 2 years.
You are caring for a family and find that the mother would like her children
vaccinated against influenza. Her children are aged 4 months, 24 months, and 5
years. Which of the following represents current immunization
recommendations for influenza?
A. None of her children should be vaccinated.
B. The 4-month-old and the 24-month-old should be vaccinated.
C. The 24-month-old and the 5-year-old should be vaccinated.
D. Only the 24-month-old should be vaccinated.
E. All the children should be vaccinated.
231. A 53-year-old man with diabetes presents to the emergency department for the
evaluation of fever. He was in his usual state of health until 36 hours ago, when
he developed fever and fatigue; these symptoms were followed by a productive
cough. Over the past few hours, he has developed worsening shortness of breath,
cough, and dizziness. He has had diabetes mellitus for many years, and he states that
he has not seen a physician in the past 12 months. On physical examination,
the patient is tachycardic and tachypnic. His blood pressure is 94/46 mm Hg, he is
orthostatic, and his temperature is 102.7° F (39.3° C). Hemoglobin O2 saturation is
88% on room air. The patient has rales and dullness to percussion at the right
pulmonary base. Chest x-ray reveals a right lower lobe infiltrate. Laboratory data
reveal a leukocytosis with left shift, as well as mild renal insufficiency. Which of
the following statements regarding community-acquired pneumonia is true? ACP
A. Bacterial pneumonia is principally spread person to person
B. The inflammatory response to Streptococcus pneumoniae or Haemophilus
influenzae often produces lobar consolidation and significant tissue necrosis
C. For patients who do not require hospitalization, advanced macrolides,
doxycycline, and respiratory fluoroquinolones are reasonable choices for therapy
D. For patients with hospital-acquired pneumonia, advanced macro- lides,
vancomycin, or doxycycline will suffice as monotherapies
233. An acutely ill child presents to the emergency department with the signs and
symptoms of acute epiglottitis. The diagnosis should be confirmed with direct
visualization of the epiglottis?
A. True
B. False
False. Routine airway visualization is stressful and may precipitate respiratory arrest. If epiglottitis is unlikely, then airway
visualization appears to be safe. In the event of respiratory arrest, laryngoscopy will be necessary for tracheal intubation.
Ahmad Soboh 56
234. A 59-year-old man with a known history of COPD presents with worsening dyspnea. On
examination, he is febrile. His breath sounds are decreased bilaterally. He is noted to
have jugular venous distension (JVD) and 2+ pitting edema of the lower extremities.
Which of the following is the most likely cause of his increasing dyspnea? Case
A. COPD exacerbation
B. Pneumonia
C. cor pulmonale
D. Pneumothorax
235. A 60-year-old man is diagnosed with moderately severe (stage II) COPD. He
admits to a long history of cigarette smoking and is still currently smoking in
counseling him about the benefits of smoking cessation, which of the
following statements is most accurate? Case-File
A. By quitting, his pulmonary function will significantly improve.
B. By quitting, his current pulmonary function will be unchanged, but the
rate of pulmonary function decline will slow.
C. By quitting, his current pulmonary function and the rate of decline are
unchanged, but there are cardiovascular benefits.
D. By quitting, his pulmonary function will approach that of a nonsmoker of
the same age.
→ Smoking cessation will not result in reversal of the lung damage that has already occurred, but can result in a
slowing in the rate of decline of pulmonary function. In fact, smoking cessation can result in the rate of decline
returning to that of a nonsmoker.
236. A 26-year-old woman presents to your office for the evaluation of fever. She was
in her usual state of health until 24 hours ago, when she developed fatigue,
myalgias, and severe headache. Her temperature rose to 102° F (38.9° C), and she
developed a nonproductive cough and chest tightness. She reports worsening
nausea and diarrhea over the same period. She denies having any sick contacts,
and she was previously healthy. On physical examination, her temperature is
101.5° F (38.6° C). Bilateral rales with tachycardia are noted. She has no
meningismus. Chest x-ray reveals bilateral, patchy air-space and interstitial
infiltrates. She is admitted to the hospital for further evaluation and monitoring.
Which of the following statements regarding Legionnaires disease is true?
A. Legionella pneumophila is typically acquired by person-to-person contact
B. There are currently no available methods of rapidly diagnosing infection
with L. pneumophila
C. Current evidence indicates that azithromycin or levofloxacin is the
treatment of choice
D. In patients in whom monotherapy with azithromycin or levofloxacin fails,
there are no other medical alternatives
Ahmad Soboh 57
238. You are treating a patient with COPD for an acute exacerbation. Assuming he
has no allergies to medications, which of the following is true regarding
antibiotic treatment in this case? Pre-Test
A. He should be prescribed amoxicillin.
B. He should be prescribed trimethoprim-sulfamethoxazole.
C. He should be prescribed doxycycline.
D. He should be prescribed azithromycin.
E. No antibiotics are necessary.
239. Which of the following medications is considered the treatment of choice for
Bordetella pertussis infection? Bratton’s
A. Azithromycin
B. Cefuroxime
C. Ciprofloxacin
D. Penicillin
E. Tetracycline
240. Which of the following is the medication of choice for the treatment of
Legionnaire's disease? Bratton’s
A. Penicillin
B. Cefuroxime
C. Azithromycin
D. Gentamicin
E. Amphotericin
241. A 12 year old male with 4 days of sore throat comes into the office. He has
been afebrile, has rhinorrhea, cough and one day of diarrhea associated with
his sore throat. The throat is mildly erythematous with otherwise normal
appearing tonsils. The best course of action is
A. Swab his throat and give a 10 day course of antibiotics, you will call him if the
culture is negative for group A strep so that he can stop antibiotic treatment.
B. Swab his throat, withhold antibiotics unless his culture is positive.
C. Give him antibiotics without testing for group A strep.
D. Symptomatic treatment
Ahmad Soboh 58
243. A 41-year-old woman presents to your office for the evaluation of fever. She was in
her usual state of health until 7 days ago, when she developed fever and severe right
facial pain. Her pain and fever have continued, and she has developed purulent nasal
drainage and foul breath odor. She admits that she has suffered from similar
symptoms in the past but never this severe. She denies having had any contact with
sick persons. Her general state of health has been good, although she has a 30 pack-
year smoking history. On physical examination, the patient’s temperature is 100.5° F
(38° C). Her pain is exacerbated when she leans forward, and there is tenderness to
palpation over the right maxillary and right frontal sinuses. Each of these sinuses is
opaque on transillumination. The rest of the examination is normal. Which of the
following statements regarding acute and chronic sinusitis is true? ACP
A. Ethmoiditis is the most common form of sinusitis in adults
B. The most useful criterion for the diagnosis of bacterial sinusitis is the
presence of purulent nasal discharge
C. Antihistamines are useful in the treatment of acute sinusitis
D. Antibiotics should be used in patients who are moderately to seriously
ill, in patients whose symptoms fail to respond to decongestants, and in
those who have complications
244. A 36-year-old day care worker presents with generalized malaise and “a cold that
won’t go away.” It started a couple of weeks ago with a runny nose and a low-grade
fever. She reports that she has now having “coughing fits,” which are sometimes so
severe that she vomits. She does not smoke and cannot remember when she last had
any immunizations. On examination, you note excessive lacrimation and conjunctival
injection. Her lungs are clear. Which one of the following is the most likely diagnosis?
A. Pertussis Bratton’s
B. Rhinovirus infection
C. Nonasthmatic eosinophilic bronchitis
D. Cough-variant asthma
E. Gastroesophageal reflux
246. Which respiration rate indicates that a child might have a pneumonia? (P.645)
A. RR 29 at age 4 years
B. RR 32 at age 25 month
C. RR 45 at age 4 years
D. RR 50 at age 2 month
247. A 45-year-old woman presents with persistent nasal congestion, facial pain
and cough for the last 10 days. She has been treated with an extended course
of nasal decongestants but despit symptoms remain. She is febrile and
otherwise looks well. Appropriate management at this time includes
A. CT scan of the sinuses
B. prescribe antibiotic
C. add nasal steroids
D. MRI of the sinuses
→ Rhinosinusitis
248. A 62-year-old man with recently diagnosed emphysema presents to your
office in November for a routine examination. He has not had any
immunizations in more than 10 years. Which of the following immunizations
would be the most appropriate for this individual? Case-File
A. Tetanus-diphtheria (Td) only
B. Tdap, pneumococcal, and influenza
C. Pneumococcal and influenza
D. Influenza only
249. A recently retired 67-year-old woman presents to you to establish care. She was a
smoker for a long time, but quit 5 years ago. She is generally healthy, but her prior
physician told her that she has “emphysema.” She was prescribed an “inhaler” to use as-
needed and only uses it rarely. She asks about necessary immunizations. Her social
history indicates that she lives with her daughter and often cares for her infant
granddaughter. Her chart indicates that she had a pneumococcal polysaccharide vaccine
at age 63 and a Td shot at age 63. Which of the following vaccines should she receive?
A. MMR
B. Tdap
C. Varicella
D. Pneumococcal polysaccharide
E. Intranasal influenza
→ The Tdap vaccine should be administered to patients 65 years and older who have close contact with an infant
aged less than 12 months.
251. A 19-year-old male presents with sore throat, fever and cervical
lymphadenopathy. A throat culture is done which is positive for group A
streptococcus. Treatment is initiated with penicillin. He returns two days later
with worsened symptoms, despite taking the medicine. Which of the following
is the most appropriate step to do next Pedia (Infect)
A. Assume the patient has infectious mononucleosis and start acyclovir and prednisone
B. Obtain a CBC and Monospot
C. Switch to azitromycin
In this case, the group A streptococcus probably represents colonization rather than the etiology of the patient's
symptoms. Infectious mononucleosis may have a similar presentation to streptococcal pharyngitis, and must be
considered if a patient is not responding clinically to treatment with antibiotics. Diagnosis may be made with a
Monospot test as well as the presence of atypical lymphocytes on CBC. EBV titers are not usually needed in diagnosis,
but may be considered if the Monospot is negative and EBV infection is to be ruled out. Treatment with acyclovir or
corticosteroids has not been proven to be of clinical benefit in uncomplicated cases of infectious mononucleosis.
252. Which of the following test may assist the clinician discriminating viral from
bacterial pneumonia? P.645
A. Sputum cytology
B. C-reactive protein
C. Chest x-ray
D. CBC
E. Procolcitonin
Ahmad Soboh 61
Additional Questions:
1. Which term is not proper for the family physician:
A. Physician specialist;
B. General profile physician;
C. Primary health care physician;
D. General practitioner.
9. In which case will you share information about the patient with your colleague?
A. Always when you are talking to colleague
B. Always when you are talking to patient's relative or family member
C. Only if talking to doctor, who takes part in medical care of the patient
D. Always, when you think that it is necessary
E. Always when you are talking to students or residents or young doctors for
educational purposes;
10. How do you think, does a patient have a right to get acquainted to medical records?
A. No, the medical records must be available only for medical persons:
B. Except very rare cases, patients must not have right to get acquainted with
medical records
C. Patients has a right to get acquainted with medical records
12. 28 years old woman admits for health evaluation. Which preventive actions
are shown in this case?
A. Ovarial palpation;
B. PAP test;
C. examination of breast;
D. All above:
E. correct answers are "2" and "3"
13. 34 years old man admits for health evaluation. Which preventive actions are
shown in this case?
A. Calculate BMI
B. Ask about tobacco use
C. Ask about alcohol use
D. Ask about Physical activities
E. All above;
14. You are making the round which is also attended by elementary course
students. How would you act when entering the room?
A. I would ask the patient's permission for the students to attend. I would
explain that it is necessary for them to develop as professionals. In case
of patient's refusal I would make the round without students.
B. I would take the students to the patient's room without agreeing with the
patient;
C. I would ask for the patient's permission about the students' attendance. I
would explain that it is necessary for them to develop as professionals. In
case of the patient's refusal I would not make the round at all, because if
we do everything for the health of patients they must help us too.
15. Which of following doesn't belong to CVD risk factors according to JNC-7 and
ESH/ESC recommendation?
A. Smoking;
B. BMI > 25;
C. Age male > 55 years; female > 65 years;
D. Diabetes mellitus;
E. C - reactive protein;
16. If blood pressure isn't elevated and patient doesn't have CVD risk factors, how
often blood pressure should be re-checked?
A. In six month;
B. In two years;
C. Annually;
D. In three years;
E. In five years.